Обсуждение:Магнитный монополь

Последнее сообщение: 3 года назад от Lesless в теме «Что-то не так: возможно пропущена буква.»

Оформление править

Под формулами не для всех обозначений приведены расшифровки

Vladimir-sergin 07:09, 23 декабря 2011 (UTC)Ответить

Правки Участника Vinograd19 править

Я закомментировал часть текста, добавленного этим участником по следующим причинам:

  1. «История» — складывалось впечатление, что в 18 веке (!) было окончательно выяснено, что монополей нет. Непонятно тогда, о чём статья.
  2. «Однако, в отличие от поля электрических зарядов и токов, поле, создаваемое магнитными зарядами, не может быть описано с помощью вектор-потенциала...» Если уравнения симметричны, то не может быть так, что для электрических зарядов может быть описано, а для магнитных — нет. Следовательно, либо это ошибка, либо пропущено что-то важное.

Требуется большая доработка и остальной части статьи. В некоторых местах просто отсутствуют логические связи между предложениями. Пример — первые же два предложения в разделе «Дираковский монополь». Извиняюсь, ошибся --SergV 21:37, 31 октября 2006 (UTC)Ответить

  1. Да, в истории нужно сделать переход от 18 века к 19. ... Извиняюсь, сейчас времени нет..
  2. Все верно. если говорить подробно (по-моему это - лишнее) - вектор потенциал вводился как rot(A)=H. Такое можно сделать только в случае div(H)=0, то есть без участия монополей.
  3. Еще хочу спросить, понятен ли вам вывод квантований Дирака? . Вот мне лично захотелось его убрать из статьи..

vinograd 22:13, 31 октября 2006 (UTC)Ответить

К сожалению, я забыл почти всё, что знал из квантовой механики. Насчёт «всё верно» не совсем понял. То есть, в случае обычных уравнений Максвела можно ввести вектор-потенциал, а в случае симметричных уравнений нельзя, так что ли? --SergV 22:41, 31 октября 2006 (UTC)Ответить
1017 ГэВ, я вижу, из английской статьи. Стоит ли, вообще, упоминать такой предел, это имеет какое-нибудь практическое значение? Звучит как муравей не может весить больше тонны. --SergV 22:47, 31 октября 2006 (UTC)Ответить
Да. Именно так. Смотрите. Классическое условие   обеспечивает соленоидальность поля H. По теореме из векторного анализа это означает, что существует такая векторная функция A, что выполняется  . То есть классическое описание полей при помощи векторного потенциала держится на теореме, одним из условий которого является  . В случае монополей оно не выполняется, а значит векторную функцию A ввести нельзя. PS. Квантовая механика тут еще ни при чем. vinograd 22:52, 31 октября 2006 (UTC)Ответить
Да, сравнение звучит не по-нашему...vinograd 22:54, 31 октября 2006 (UTC)Ответить
А, теперь я кажется понял. В том абзаце говорилось только о магнитном поле, не об электрическом? Если магнитное поле создаётся только электрическими зарядами, то его можно описать векторным потенциалом, а если ещё и магнитными, то нельзя. --SergV 23:23, 31 октября 2006 (UTC)Ответить
Нет. Еще раз:
  1. Если существуют только электрические заряды, то Электромагнитное (и E и B) описывается векторным потенциалом.
  2. Если существуют только магнтные заряды, то Электромагнитное (и E и B) описывается векторным потенциалом.
  3. Если существуют только и магнтные заряды электрические заряды, то Электромагнитное (и E и B) не описывается векторным потенциалом.
vinograd 09:57, 1 ноября 2006 (UTC)Ответить
Тогда, значит, четырёхмерным, включающим скалярный электрический потенциал? Теперь понятно, что значат цифры в скобках. --SergV 18:56, 1 ноября 2006 (UTC)Ответить

Система физических величин править

Магнитный источник состоит из четырёхмерной 2-формы магнитного тока   и четырёхмерной 3-формы плотности магнитного заряда  :

 
 

По теореме де Рама магнитный четыре ток можно представить через вторую форму:

 

Это обобщение от:

 

При постоянных магнитных источниках   не может быть выражена через 1-форму   для четырёхмерного потенциала электромагнитного поля (используем теорему де Рама, лемму Пуанкаре):

 
 

В случае электрических и магнитных источников имеем:

 
 

Или что тоже самое:

 
 
 
 

Четыре 2-Формы и Четыре источники (3-формы) взаимно заменяемы (можно трансформировать одно в другое):

 

При исчезающих электрических источниках (   )   может быть выажена через магнитный четыре потенциал:

 

И соответственно:

 

В случае наличия электрических и магнитных источников: Поля   появляются из электрических источников   и определяются через электрический четыре потенциал  . Поля   появляются из магнитных источников   и определяются через магнитный четыре потенциал  .

ignat

Игант, можно в двух словах, Что и Зачем вы сказли? vinograd 16:47, 1 ноября 2006 (UTC)Ответить
Мне не нравиться, как люди путаются, и я решил дать точный ответ на Ваш вопрос. ignat 08:09, 2 ноября 2006 (UTC)Ответить
Извините Игнат, но я еще не знаком с системой величин Плотниккова. Думаю, что с ней мало кто знаком в принципе. Ссылки на нее не надо включать в статьи, где можно разобраться класическими методами. Главное - правильно использовать инструмент - русский язык... Может быть вы составите статью про эту систему для "Начинающих"? vinograd 08:37, 2 ноября 2006 (UTC)Ответить
Классическими это циркулем и линейкой :-) ??? Ловите в личной почте ссылки на литературу. Впрочем в статье Система физических величин так же есть внизу ссылки (Вы могли бы быть и повнимательнее - первая ссылка ключевая). То что я привожу, это общепринятая в мире нотация для такого рода задач. Можете спросить у своего преподавателя. ignat 12:12, 2 ноября 2006 (UTC)Ответить
"Классическим - это циркулем и линейкой" - согласен. Энциклопедия должна быть общедоступной. Статьи надо писать так, чтобы читатель остановился на чтении там, где перестаёт что-либо понимать, при этом в голове должен остаться четко изложенный материал. Предлагаю вам сделать секцию в этой статье, где вы напишите всё, что считаете нужным. vinograd 13:45, 2 ноября 2006 (UTC)Ответить

Вопрос!!! править

"Магнитный монополь — частица, обладающая «магнитным зарядом» — точечным источником радиального магнитного поля."

Насколько я знаю, магнитное поле(как часть электромагнитного) создаётся только движущимися зарядами. В соответствии с постулатом Эйнштейна, если в одной инерциальной системе отсчёта существует магнитное поле, то в другой оно может вообще не существовать!!! Следует ли из факта существования магнитного монополя, или отрицание постулата Эйнштейна, или отрицание взаимосвязи электрического и магнитного поля в электромагнитное(т. е. отрицание уравнений Максвелла) ?????

P. S. Если я не прав в рассуждениях, объясните, пожалуйста, где...

Gvozdet 11:07, 15 июля 2009 (UTC)Ответить
Хороший вопрос! Странно, что на него до сих пор никто НЕ обратил внимание... особенно в связи с т.н. "открытием" последнего в спиновых стеклах. А ведь с точки зрения логики и здравого смысла на него существует единственный ответ - "магнитный монополь" имеет размеры, равные размерам нашей Вселенной, и поэтому НЕ существует ДРУГОЙ инерциальной системы, в рамках которой наш движущийся электрический заряд (создающий монополь), ПОКОИТСЯ!

PS: Подлинный магнитный монополь должен иметь "градиентное" (потенциальное) магнитное поле, подобно до электрического заряда.195.47.212.108 06:17, 12 января 2010 (UTC)Ответить

Если я правильно понял, если предположить, что магнитный монополь имеет размеры, равные размерам Вселенной и имеет потенциальное магнитное поле, то, вероятно, неоднородности потенциального поля являются магнитными диполями. Таким образом, диполь, это частный случай (аномалия?)магнитного монополя??? Putnik178 10:01, 20 января 2013 (UTC)Ответить

Предполагать это не надо статья не о монополях конечного размера. АИ или не АИ? Alexander Mayorov 12:10, 20 января 2013 (UTC)Ответить
Поле с такой радиальной конфигурацией не может обнулиться и в какой инерциальной системе отсчёта (согласно уравнениям электродинамики Максвелла). Что касается магнитного поля электрических проводников с током, в которых электрические заряды одного знака движутся относительно зарядов противоположного знака, то, ни в какой системе отсчёта, скорости движения электрических зарядов в таких электрически нейтральных проводниках не могут все быть нулевыми. 89.222.164.159 07:45, 16 марта 2016 (UTC)Ответить

Что-то не так: возможно пропущено слово. править

волновая функция  , характеризующая частицу с зарядом   при изменении   как  .

Что-то не так: возможно пропущена буква. править

Комплексное число равно единице, если оно предствлено 104.162.239.6 22:00, 12 января 2021 (UTC)Ответить